linzru86
Thanks Received: 3
Forum Guests
 
Posts: 24
Joined: June 08th, 2010
 
 
 

Q9 - Since anyone who supports

by linzru86 Mon Aug 02, 2010 1:38 pm

These kinds of questions usually trip me up and this one is no exception. Could you clear it up for me? Thanks!
User avatar
 
ManhattanPrepLSAT1
Thanks Received: 1909
Atticus Finch
Atticus Finch
 
Posts: 2851
Joined: October 07th, 2009
 
This post thanked 9 times.
 
 

Re: Q9 - Since anyone who supports

by ManhattanPrepLSAT1 Wed Aug 04, 2010 2:43 pm

This is a great question! I have a particular fondness for ones that play on conditional logic...

We're asked to figure out where this argument goes wrong. The key here is to find the assumption and then attack it.

Evidence

1. Anyone who supports the new tax plan has no chance of being elected.
2. Anyone who truly understands economics would not support the tax plan.


Conclusion

Only someone who truly understands economics would have any chance of being elected.

Put into formal notation...

STP ---> ~E
UE ---> ~STP
========
E ---> UE

(Formal Notation Key: STP = supports the tax plan, E = elected, UE = understands economics)

If we take the contrapositive of the first conditional statement

E ---> ~STP
UE ---> ~STP
========
E ---> UE

We can see that the flaw in this argument is that it commits a reversal of the second premise. It should say ~STP ---> UE, so that's the assumption. To weaken this argument we attack the assumption by meeting the sufficient condition and failing to meet the necessary condition. So the correct answer should say something like ~STP some ~UE. Put into English, "some people who do not support the new tax plan do not truly understand economics." We see this best stated in answer choice (D).

(A) reinforces the assumption rather than attacks the assumption.
(B) sidesteps the conclusion. The conclusion is about those who have a chance of being elected, not those who don't.
(C) sidesteps the conclusion. The conclusion is about those who have a chance of being elected, not those who don't.
(E) sidesteps the conclusion. The conclusion is about those who have a chance of being elected, not those who don't.

The work is all done up front. Find what you think the correct choice should say, and then scan the answer choices looking for what you believe will attack the assumption of the argument!

I hope this helps. Let me know if you still need some more help seeing this one...
 
peg_city
Thanks Received: 3
Forum Guests
 
Posts: 152
Joined: January 31st, 2011
Location: Winnipeg
 
 
trophy
First Responder
 

Re: Q9 - Since anyone who supports

by peg_city Mon May 16, 2011 3:10 pm

I got all the inferences right.

I'm still stuck on (D).

Doesn't it read ~STP -> ~UE

"People who do not support the tax plan do not truly understand economics"
User avatar
 
ManhattanPrepLSAT1
Thanks Received: 1909
Atticus Finch
Atticus Finch
 
Posts: 2851
Joined: October 07th, 2009
 
This post thanked 1 time.
 
 

Re: Q9 - Since anyone who supports

by ManhattanPrepLSAT1 Thu May 26, 2011 11:48 pm

peg_city Wrote:I'm still stuck on (D).

Doesn't it read ~STP -> ~UE


Close, but not exactly. Answer choice (D) is a quantified statement, not a conditional one. And part of the statement is contained in the question stem. In the question stem it says that "the argument ignores the possibility that some people who..."

So the "some" is stated in the question stem. And answer choice (D) should be represented as

~STP some ~UE

Does that answer your question?
 
peg_city
Thanks Received: 3
Forum Guests
 
Posts: 152
Joined: January 31st, 2011
Location: Winnipeg
 
 
trophy
First Responder
 

Re: Q9 - Since anyone who supports

by peg_city Mon May 30, 2011 1:15 pm

yes, thanks very much :D :D
 
Daniella.owusu
Thanks Received: 5
Vinny Gambini
Vinny Gambini
 
Posts: 12
Joined: December 04th, 2011
 
 
 

Re: Q9 - Since anyone who supports

by Daniella.owusu Mon Feb 27, 2012 7:38 pm

I'm really confused with this question!!

The first premise reads:
STP--->> ~E
Contrapositive: E--->> ~STP

second premise:
UE--->> ~STP
contrapositive: STP--->> ~UE

Conclusion:
E--->> UE
~UE--->> ~E

I just don't understand how the flaw is D. I'm really not sure about any of the answers so if someone can really explain in a simpler way for me to comprehend
 
timmydoeslsat
Thanks Received: 887
Atticus Finch
Atticus Finch
 
Posts: 1136
Joined: June 20th, 2011
 
This post thanked 7 times.
 
trophy
Most Thanked
trophy
First Responder
 

Re: Q9 - Since anyone who supports

by timmydoeslsat Mon Feb 27, 2012 11:40 pm

So, we can agree that the premises and conclusion look like this:

Chance Elected ---> ~Support
Understand Econ ---> ~Support
______________________________
Chance Elected ---> Understand Econ


I think the confusion on this question is in the question stem itself. We are not being asked for a sufficient assumption. In this case, the argument is assuming:

~Support ---> Understand Econ

If you are having trouble how I spotted this assumption, then let us first look to the conclusion. It is a conditional statement.

The sufficient condition is Chance Elected. Its necessary condition is Understand Econ. We are given only one statement regarding Chance Elected, the first premise of the argument. It ends up with ~Support. The argument must somehow go from this idea of ~Support to the idea of Understand Econ that is given as the necessary condition in the conclusion.

So, the argument assumes that ~Support ---> Understand Econ.

Since we now see that this is what the argument assumes, we can attempt to figure out what the argument is not considering. It will be addressing this assumption. Do we really know that if you are ~Support, that you will necessarily Understand Econ? We do not.

If we can show that some ~Support do not Understand Econ, we will have shown how this assumption was flawed.

That is what (D) does. It says ~Support SOME ~Understand Econ.


That is the formal logic approach to this question. Another is to take a more intuitive approach of reasoning.

The first sentence tells us that anyone who supports the new tax plan has no chance of being elected.

So, we know right off the bat that if you want to be elected, you cannot be for the new tax plan.

Next sentence, if you understand econ, then you will not support it.

Ok, so as of now, it reasons that people that understand econ are not automatically out of contention for them having a chance at the election. They are not supporting the tax plan, which is a requirement for having a chance. So, they are still in the running.

Yet, the conclusion reasons that it is necessary to understand econ to have a chance at the election.

What about people that do not understand econ? They may also be candidates that do not support the new tax plan. So, we can show that these people would challenge the idea that understanding econ is necessary for having a chance at the election.
User avatar
 
ManhattanPrepLSAT1
Thanks Received: 1909
Atticus Finch
Atticus Finch
 
Posts: 2851
Joined: October 07th, 2009
 
 
 

Re: Q9 - Since anyone who supports

by ManhattanPrepLSAT1 Fri Mar 02, 2012 7:33 pm

Timmy's explanation is great! Let me just add a more general idea that may help.

Suppose I assume in an argument that "all 6th graders enjoy reading." How would I undermine an argument with such an assumption? I'd attack the argument by saying the assumption may not be true. What's the logical opposite of "all 6th graders enjoy reading?" -- "Not all 6th graders enjoy reading."

Or in other words, "some 6th graders do not enjoy reading."

In this argument, it is assumed that, "anyone who does not support the tax plan understands economics." How do we counter this assumption? We say that the argument failed to consider the possibility that "some people who do not support the tax plan do not understand economics."

Hope that helps!
 
shaynfernandez
Thanks Received: 5
Elle Woods
Elle Woods
 
Posts: 91
Joined: July 14th, 2011
 
 
 

Re: Q9 - Since anyone who supports

by shaynfernandez Thu Jul 05, 2012 11:52 am

The hardest part of this problem for me was the question stem. I was so confused by it that I not only read it before the stimulus but a few times after and throughout attacking the answers. Making this question extremely time consuming early on in the test. What I understand from the stem is that:
1. It's flawed
2. We are asked to show what was not considered and thus weakening the argument.
The stimulus and answer choices also
3. display conditional logic throughout
4. We can see that we have a gap that needs to be filled with an assumption.

So, my question is what category would we put this question in?
At first glance I labeled it as a flaw question, then a weakening, we have the same process for each but how do we know exactly what we are looking for in order to weed out the irrelevant answers?
User avatar
 
WaltGrace1983
Thanks Received: 207
Atticus Finch
Atticus Finch
 
Posts: 837
Joined: March 30th, 2013
 
 
trophy
Most Thanked
trophy
Most Thankful
trophy
First Responder
 

Re: Q9 - Since anyone who supports

by WaltGrace1983 Fri Jun 20, 2014 3:03 pm

I'm going to rehash this problem. I always (and I mean always) have a difficult time with this problem. This is one of those that I saw at the beginning of my prep and I return to it every month or two just to see if anything has changed. Usually I end up in the same place - completely confused. However, today I was able to solve it much easier because I have simply been working on understanding how important the actual question/start of an incorrect answer choice is. What I mean is that I have been trying to focus on what "ignores the possibility" or "takes for granted" really means. I think that led me down the right path today and I wanted to do a write-up; maybe it will help someone else!

Key:
    "Supports the new tax plan" = STP
    "No chance at being elected" = ~E
    "Truly understands economics" = UE


The Core:
    (E→~STP)
    +
    (UE→~STP)
    →
    E → UE


Let's think about this. Can we chain up any conditionals? No, it doesn't seem so.

However, let's just think about this question in the abstract. Isn't it a REALLY strong conclusion, to say that ONLY those who are (UE) have a chance at being elected, (E)? Couldn't other people get elected? This is the approach I went with.

What do we know about (E)? We know that (E) → ~(STP). Thus, we know that whoever has a chance at being elected, (E), must be ~(STP). We know that (UE) is sufficient for us to get there. But what about ~(UE)? We know nothing about ~(UE) when it is the sufficient condition. Couldn't it be that people are ~(UE) can also get elected? We have no reason not to believe so!

That is why (D) is correct.
User avatar
 
Mab6q
Thanks Received: 31
Atticus Finch
Atticus Finch
 
Posts: 290
Joined: June 30th, 2013
 
 
 

Re: Q9 - Since anyone who supports

by Mab6q Sun Feb 08, 2015 3:00 pm

Tricky question. I got this one wrong, not because I didn't understand the logic, but because I misread the question stem.

The question requires us to begin all of the answer choices with some, something I failed to notice. If not for that, B could also be correct.

It would be UE -- > ~E , which would give us the contrapositive: E --> ~UE.

I just wanted to make sure my logic was right on that if someone can confirm? Of course this is under the hypo that we take the some out of the question stem.
"Just keep swimming"
User avatar
 
ohthatpatrick
Thanks Received: 3808
Atticus Finch
Atticus Finch
 
Posts: 4661
Joined: April 01st, 2011
 
 
 

Re: Q9 - Since anyone who supports

by ohthatpatrick Thu Feb 12, 2015 2:11 pm

Correct, (B), if it were ALL, would read
Understand -> ~Chance
Chance -> ~Understand
and the conclusion was
Chance --> Understand

Those two definitely contradict each other. It should seem weird as an answer, though, since it doesn't seem very LSAT like to have an answer just flat out contradict the conclusion, with no reference at all to what was mentioned in the premises.

A lot of people have mentioned the question stem confusion. The keyword "flaw" means it's a Flaw question.

However, when Flaw answer choices start with "takes for granted / presumes", we read them like Necessary Assumption answers.

And when Flaw answer choices start with "fails to consider / ignores the possibility", we read them like Weaken answers.

So since the question stem pre-loads all the answer choices with "ignores the possibility", we can just read this like Weaken.

PREMISES:
Support -> ~Chance
Understands --> ~Support

CONC:
Chance -> Understands

Fairly straightforward to see the author is trying to combine to conditional premises to get to his conclusion. But, of course, the premises do NOT chain. Because this is Flaw, we should get into the author's head and figure out what he was thinking.

Where did he think we got from
Chance ->
?

We know that
Chance -> ~Support

He thinks we get all the way from
Chance ------------------> Understands

So he's THINKING we have this link
Chance -> ~Support --> Understands

But of course, that was backwards from the connection we were really given.

If the answer choices were written descriptively, we'd expect this Flaw to be described as the Necessary/Sufficient Flaw, i.e. botching conditional logic.

Instead, we need to figure out how to provide a counterexample to the author's FLAWED link.

Remember, his bad connection is
~Supports --> Understands
(or equivalently)
~Understand --> Supports

To provide a counterexample to a conditional, you need something that IS the Sufficient but ISN'T the Necessary.

So we either need an example of someone who is ~Support and ~Understand, which is (D).
 
yunjh2725
Thanks Received: 0
Vinny Gambini
Vinny Gambini
 
Posts: 5
Joined: September 25th, 2014
 
 
 

Re: Q9 - Since anyone who supports

by yunjh2725 Fri Sep 18, 2015 7:34 pm

Can someone check my method:

B: Some people who understand econ have no chance of being elected.
(Since they understand econ, they must not support the tax plan. But, what do we know about those who do not support the tax plan? We only know about those who support tax plan have no chance of being elected. How will this weaken the conclusion that just says that it is necessary to understand to have chance of being elected? We have to find an answer that says something about you don't have to understand economics to have a chance of being elected. Maybe there are those who can still win without understanding)

E: Some have no chance of being elected do not truly understand econ.
(Same as above. We don't know anything about those who have a chance of being elected without satisfying the requirement)

D: Those who do not support the tax plan do not truly understand econ.
(This weakens the conclusion because there are those who do not understand econ but still does not support the tax plan. So they have a equal chance of being elected. It weakens the argument by saying that "no, you do not HAVE to understand econ to not support the tax plan and lose the chance of being elected)
 
ellylb
Thanks Received: 0
Vinny Gambini
Vinny Gambini
 
Posts: 11
Joined: March 29th, 2016
 
 
 

Re: Q9 - Since anyone who supports

by ellylb Tue Apr 12, 2016 5:28 pm

Hey,
I have a v simple formal diagram, does this work?

E - > /STP AND UE

So not supporting the new tax plan AND understanding economics are the two necessary conditions for a chance of being elected. Therefore one only need attack one such necessary condition to make the statement fail. This is done when we note that some people who don't support the tax plan (/STP) actually do not understand economics either, meaning that the two necessary conditions needed for election would not be filled
 
hwangbo.edu
Thanks Received: 2
Vinny Gambini
Vinny Gambini
 
Posts: 15
Joined: April 24th, 2016
 
 
 

Re: Q9 - Since anyone who supports

by hwangbo.edu Sun Apr 24, 2016 5:54 pm

elica.zadeh Wrote:Hey,
I have a v simple formal diagram, does this work?

E - > /STP AND UE



No, this diagram doesn't represent the argument. Check the stimulus carefully: the premises establish no relationship between E and UE, yet the conclusion mistakenly assumes that /STP --> UE.

mattsherman (2d post) explains the flawed argument in detail.
 
fadams
Thanks Received: 4
Vinny Gambini
Vinny Gambini
 
Posts: 19
Joined: July 21st, 2014
 
This post thanked 2 times.
 
 

Re: Q9 - Since anyone who supports

by fadams Thu Jun 23, 2016 3:59 pm

To simplify:

A (elected) -> B (not support)
C (understand) -> B (not support)
____________________________________
A (elected) -> C (understand)

Form:
A->B
C->B
_____
A->C


The assumption is
B -> C
Because if that is explicitly stated, you can chain the premises together into the classic form
A -> B -> C; Thus A->C.

So in this case, B->C translates to
not support -> understand

To attack this assumption, as you would just like in weaken questions that involve a conditional assumption, you fail the necessary condition to say that whatever is necessary is not for the sufficient to occur.

Hence the answer D, says
not support -> not understand
is correct.
 
kmcaple
Thanks Received: 2
Vinny Gambini
Vinny Gambini
 
Posts: 2
Joined: September 10th, 2016
 
 
 

Re: Q9 - Since anyone who supports

by kmcaple Mon May 15, 2017 2:35 pm

Vinny Gambini: this is the most understandable approach to this question for me. It has baffled my brain for the last two hours.

I could not understand how -s -> -u when to be correct it would need to read s -> -u. When you explained that I needed to weaken their conclusion, and one way to do that was to say the necessary did not have to be necessary, it clicked. Thanks so much!
 
KyleeK529
Thanks Received: 0
Vinny Gambini
Vinny Gambini
 
Posts: 1
Joined: February 09th, 2018
 
 
 

Re: Q9 - Since anyone who supports

by KyleeK529 Mon Mar 19, 2018 2:38 am

Hello,

I actually have a question regarding the conclusion of this conditional logic question.

I do not understand why the conclusion is [electable --> understand economics], and NOT [understand economics --> electable].


Thank you very much!
 
LindaJ180
Thanks Received: 0
Vinny Gambini
Vinny Gambini
 
Posts: 1
Joined: September 02nd, 2018
 
 
 

Re: Q9 - Since anyone who supports

by LindaJ180 Sun Sep 02, 2018 2:00 pm

how do you know which statement to take the contrapositive and which assumption to attack. how did you know to take the contrapositive of the first statement. I would like to know so that i could apply this to other questions. Thank you
 
AnnaT620
Thanks Received: 0
Elle Woods
Elle Woods
 
Posts: 51
Joined: May 25th, 2020
 
 
 

Re: Q9 - Since anyone who supports

by AnnaT620 Sat Jul 18, 2020 12:45 pm

Hi MP Team

From reading the above and the textbook, I understand that the argument is attempting to use the contrapositive of the first statement to link up with the second statement. I was able to get to the point where you can see that the premises don't validly link up into a chain. I'm just not sure I would have understood how to make that connection as to where the author of the conclusion went wrong by myself. And even once you've made that, how you get to the assumption "anyone who does not support the tax plan, does understand economics" and the correct answer in D.

I had originally picked A because it mirrored the premises, but my understanding is that's wrong because it simply mirrors the premise and does not invalidate the argument. I just am struggling with the idea that you're picking an answer choice that invalidates the argument, when the argument is already invalid?

Many thanks!
Anna